Administración     

Olimpiadas de Matemáticas
Página de preparación y problemas

Selector
La base de datos contiene 1154 problemas y 775 soluciones.
OME Local
OME Nacional
OIM
OME Andalucía
Retos UJA
Problema 360
Dados números reales $a,b,c\gt 0$, demostrar que \[\left(1+\frac{a}{b}\right)\left(1+\frac{b}{c}\right)\left(1+\frac{c}{a}\right)\geq 2\left(1+\frac{a+b+c}{\sqrt[3]{abc}}\right).\]
pistasolución 1info
Pista. Haz el cambio $x=\frac{a}{\sqrt[3]{abc}}$, $y=\frac{b}{\sqrt[3]{abc}}$, $z=\frac{c}{\sqrt[3]{abc}}$, con lo que desaparece la raíz y queda la condición $xyz=1$. Después manipula la desigualdad resultante para expresarla de la forma más sencilla posible.
Solución. En primer lugar, haciendo el cambio de variables \[x=\frac{a}{\sqrt[3]{abc}},\ y=\frac{b}{\sqrt[3]{abc}},\ z=\frac{c}{\sqrt[3]{abc}},\] tenemos la desigualdad equivalente \[\left(1+\frac{x}{y}\right)\left(1+\frac{y}{z}\right)\left(1+\frac{z}{x}\right)\geq 2(1+x+y+z),\] es decir, nos hemos deshecho de la raíz cúbica, a cambio de la condición $xyz=1$. Ésta nos permite reescribir la siguiente desigualdad anterior como \[(x+y)(y+z)(x+z)\geq 2(1+x+y+z).\] Ahora bien, teniendo en cuenta las identidades \begin{eqnarray*} (x+y)(y+z)(x+z)&=&2xyz+(x^2y+xy^2+x^2z+zx^2+y^2z+z^2y),\\ (x+y+z)(xy+yz+xz)&=&3xyz+(x^2y+xy^2+x^2z+zx^2+y^2z+z^2y), \end{eqnarray*} y que $xyz=1$, deducimos $(x+y)(y+z)(x+z)=(x+y+z)(xy+yz+xz)-1$. Así podemos reescribir de nuevo la desigualdad a probar como \[(x+y+z)(xy+yz+xz-2)\geq 3.\] Esta última desigualdad se deduce de aplicar la desigualdad entre las medias aritmética y geométrica a las ternas $(x,y,z)$ y $(xy,yz,xz)$, es decir, de las desigualdades elementales \[x+y+z\geq 3\sqrt[3]{xyz}=3,\qquad xy+yz+xz\geq 3\sqrt[3]{x^2y^2z^2}=3.\]
Si crees que el enunciado contiene un error o imprecisión o bien crees que la información sobre la procedencia del problema es incorrecta, puedes notificarlo usando los siguientes botones:
Informar de error en enunciado Informar de procedencia del problema
Problema 359
Sea $P(x)$ un polinomio con coeficientes enteros que es igual a $1979$ para cuatro valores enteros de $x$ distintos. Demostrar que $P(x)$ no es igual a $2\cdot 1979$ para ningún valor entero de $x$.
pistasolución 1info
Pista. Usa que $a-b$ divide a $P(a)-P(b)$ para cualesquiera enteros distintos $a$ y $b$. También viene bien saber que $1979$ es un número primo (probablemente algo que sabían quienes participaron aquel año).
Solución. Pongamos que $P(x_0)=2\cdot 1979$ y $P(x_1)=P(x_2)=P(x_3)=P(x_4)=1979$. Usando la conocida propiedad para polinomios de coeficientes enteros que dice que $a-b$ divide a $P(a)-P(b)$ para cualesquiera números enteros $a$ y $b$ distintos, obtenemos que $x_i-x_0$ divide a $1979$ para $i=1,2,3,4$. Como $x_1,x_2,x_3,x_4$ son distintos y $1979$ es primo, esto no nos deja otra opción que $x_1-x_0=1$, $x_2-x_0=-1$, $x_3-x_0=1979$ y $x_4-x_0=-1979$, salvo renombrar las variables. De hecho, podemos aplicar una traslación en $x$ para suponer que $x_1=-1979$, $x_2=-1$, $x_0=0$, $x_4=1$ y $x_5=1979$ sin perder generalidad.

Si escribimos $p(x)=a_0+a_1x+\ldots+a_n$, entonces $p(0)=a_0=2\cdot 1979$. Por otro lado, \begin{align*} p(1979)&=a_0+1979a_1+1979^2a_2+\ldots+1979^na_n\equiv a_0+1979a_1\ (\text{mod }1979^2),\\ p(-1979)&=a_0-1979a_1+1979^2a_2+\ldots+(-1979)^na_n\equiv a_0-1979a_1\ (\text{mod }1979^2). \end{align*} Sumando estas dos congruencias, tenemos que $2a_0\equiv p(1979)+p(-1979)=2\cdot 1979\ (\text{mod }1979^2)$, pero esto es una contradicción ya que $2a_0\equiv 4\cdot 1979\not\equiv2\cdot 1979\ (\text{mod }1979^2)$.

Si crees que el enunciado contiene un error o imprecisión o bien crees que la información sobre la procedencia del problema es incorrecta, puedes notificarlo usando los siguientes botones:
Informar de error en enunciado Informar de procedencia del problema
Problema 358
Sea $ABC$ un triángulo con $\angle A=90$ y $\angle B\lt\angle C$. La tangente en $A$ a la circunferencia $\Gamma$ circunscrita a $ABC$ corta a la recta $BC$ en $D$. Sean $E$ el punto simétrico de $A$ respecto de la recta $BC$, $X$ el pie de la perpendicular desde $A$ a la recta $BE$ e $Y$ el punto medio de $AX$. Si la recta $BY$ corta de nuevo a $\Gamma$ en un punto $Z$, demostrar que la recta $BD$ es tangente a la circunferencia circunscrita del triángulo $ADZ$.
Sin pistas
Sin soluciones
info
Si crees que el enunciado contiene un error o imprecisión o bien crees que la información sobre la procedencia del problema es incorrecta, puedes notificarlo usando los siguientes botones:
Informar de error en enunciado Informar de procedencia del problema
Problema 357
Dados enteros positivos $m,n\in\mathbb{N}$ no ambos iguales a $1$ y reales positivos $x,y\in\mathbb{R}_+$, demostrar que \[(n-1)(m-1)(x^{n+m}+y^{n+m})+(n+m-1)(x^ny^m+x^my^n)\geq nm(x^{n+m-2}y^2+x^2y^{n+m-2}).\]
pistasolución 1info
Pista. Aplica la desigualdad de Jensen en $[0,n]$ con pesos a la función convexa $f(t)=x^{m+n}(\frac{y}{x})^t+y^{m+n}(\frac{x}{y})^t$.
Solución. Supongamos que $x$ e $y$ son números fijos a lo largo del siguiente razonamiento y que $x\neq y$ (ya que en tal caso, la desigualdad es obviamente una igualdad). Además, supondremos que $m\geq n$ sin perder generalidad. Definimos una función $f:\mathbb{R}\to\mathbb{R}$ (a la que aplicaremos la desigualdad de Jensen) como \[f(t)=x^{m+n}(\tfrac{y}{x})^t+y^{m+n}(\tfrac{x}{y})^t.\] Esta función es derivable y su derivada viene dada por \[f'(t)=x^{m+n}(\tfrac{y}{x})^t\ln(\tfrac{y}{x})+y^{m+n}(\tfrac{x}{y})^t\ln(\tfrac{x}{y})=x^ty^t(x^{m+n-2t}-y^{m+n-2t})\ln(\tfrac{y}{x}).\] El factor $(x^{m+n-2t}-y^{m+n-2t})$ tiene el signo opuesto a $\ln(\tfrac{y}{x})$ para $t\in[0,\frac{m+n}{2}]$ y el mismo signo si $t\in[\tfrac{m+n}{2},m+n]$. Por lo tanto, se sigue que $f$ es estrictamente decreciente en $[0,\frac{m+n}{2}]$ y estrictamente creciente en $[\tfrac{m+n}{2},m+n]$ (siempre que $x\neq y$, ya que $f$ es constante si $x=y$). La segunda derivada de $f$ viene dada por \[f''(t)=x^{m+n}(\tfrac{y}{x})^t\ln(\tfrac{y}{x})^2+y^{m+n}(\tfrac{x}{y})^t\ln(\tfrac{x}{y})^2,\] que es claramente positivo (de nuevo usamos que $x\neq y$), en cuyo caso tenemos que $f$ es una función convexa. La desigualdad de Jensen con pesos nos dice que $$af(u)+bf(v)\geq(a+b)f\left(\frac{au+bv}{a+b}\right)$$ para cualesquiera $a,b\gt 0$ y $u,v\in\mathbb{R}$. Tomando $a=(n-1)(m-1)$ y $b=m+n-1$, obtenemos que $a+b=mn$. Elegimos también $u=0$ y $v=n$, lo que nos dice que el miembro de la izquierda es el del enunciado: $$af(u)+bf(v)=(n-1)(m-1)(x^{m+n}+y^{m+n})+(m+n-1)(x^my^n+x^ny^m).$$ Por su parte, el miembro de la derecha en Jensen podemos estimarlo para $m+n\geq 3$ como $$(a+b)f(\tfrac{au+bv}{a+b})=nm f(1+\tfrac{n-1}{m})\geq nm f(2)=nm(x^{m+n-2}y^2+x^2y^{m+n-2}),$$ donde hemos usado que $f$ es decreciente en $[0,\frac{m+n}{2}]$ y simétrica respecto de $x=\frac{m+n}{2}$. Nos queda así por analizar el caso $(m,n)=(2,1)$, ya que estamos suponiendo $m\geq n$ y $(m,n)=(1,1)$ está descartado por el enunciado (en este caso, la desigualdad se reduce a $-(x-y)^2\geq 0$ y no es cierta). Sin embargo, al sustituir $m=2$ y $n=1$ en el enunciado, obtenemos $0\geq 0$, luego en este caso siempre se da la igualdad.

Nota. Si $x=y$ o bien $(m,n)=(2,1)$ o $(m,n)=(1,2)$, se tiene la igualdad, como ya se ha comentado. No hay más casos en que se dé la igualdad, ya que Jensen nos dice que $a=0$ o $b=0$ ya que $f$ es estrictamente convexa y $u\neq v$. Como quiera que $b\geq 1$, tenemos necesariamente que $a=0$, luego $m=1$ o $n=1$. Además, se tiene que dar que $f(1+\frac{n-1}{m})=f(2)$, lo que nos lleva a que $m+n=3$ y, por tanto, $m=2$ y $n=1$.

Si crees que el enunciado contiene un error o imprecisión o bien crees que la información sobre la procedencia del problema es incorrecta, puedes notificarlo usando los siguientes botones:
Informar de error en enunciado Informar de procedencia del problema
Problema 356
Sea $P(x)$ un polinomio con coeficientes enteros no negativos y supongamos que $m\geq 2$ es un entero tal que $\sqrt[m]{P(n)}$ es racional para todo $n\in\mathbb{N}\cup\{0\}$. Demostrar que existe un polinomio $Q(x)$ con coeficientes enteros tal que $P(x)=Q(x)^m$ para todo $x\in\mathbb{R}$
Sin pistas
Sin soluciones
info
Si crees que el enunciado contiene un error o imprecisión o bien crees que la información sobre la procedencia del problema es incorrecta, puedes notificarlo usando los siguientes botones:
Informar de error en enunciado Informar de procedencia del problema
José Miguel Manzano © 2010-2024. Esta página ha sido creada mediante software libre